Вы находитесь на странице: 1из 10

Introducao Analise Funcional

Lista de excercicios
Aluno: Segundo Manuel Argomedo Salirrosas

20. Seja X um espaco normado. Mostre:


(a) toda sequencia convergente em X e de Cauchy;
(b) toda sequencia de Cauchy em X e limitada;
(c) se uma sequencia de Cauchy admitir uma subsequencia convergente, entao a
propria sequencia e convergente, convergindo para o mesmo limite da subsequencia
Soluc
ao:
(a) Seja xn x, dado > 0, existe n0 N tal que n n0 , kxn xk < /2.
Sejam m, n 0,
kxm xn k = kxm x + x xn k kxm xk + kxn xk < /2 + /2 =
Portanto (xn ) e uma sequencia de Cauchy.
(b) Consideremos = 1, pelo jeito de ser de Cauchy existe n0 tal que se m, n n0
tem-se que kxn xm k < 1 do jeito tal que se n > n0 , xn B1 (xn0 +1 ). Somente
um n
umero finito de termos ficaram fora desta bola.
Seja r = max{kx1 xn0 k, . . . , kxn0 xn0 +1 k}.
Para todo n temos que kxn xn0 +1 k r. Assim
{xn : n = 1, . . . , } Br+1 (xn0 +1 )
(c) Seja (xn ) uma sequencia de Cauchy. Como (xni ) y temos que dado > 0
existe i0 N tal que para i > i0 , kxni yk < /2. Logo:
kxn yk = kxn xni + xni yk kxn xni k + kxni yk < /2 + /2 =
23. Sejam X, Y , Z espacos normados e K Z um conjunto compacto. Dada uma
aplicacao contnua f : X K Y e fixado x0 X, mostre que para todo > 0
existe > 0 tal que
x X, kx x0 k <

kf (x, t) f (x0 , t)k < , t K.

Soluc
ao:
Sejam (xn ) X e (tn ) K dois sequencias e seja xn x0 .
Suponhamos que dado > 0 existe = 1/n tal que se,
kxn x0 k <

1
n

kf (xn , t) f (x0 , t)k


1

Como K e compacto, temos que tni t K e do fato que toda subsequencia de


uma sequencia convergente converge para o mesmo limite, isto e xni x0 , temos que
lim f (xni , tni ) = f (x0 , t)

Se tni = t, sequencia constante entao kf (xni , t) f (x0 , t)k 0 ().


30. Sejam X um espaco normado e A X um subconjunto nao vazio. Mostre que
|dist(x0 , A) dist(y0 , A)| kx0 y0 k
para quaisquer x0 , y0 X. Assim, a funcao dist(, A) : X R+ e uniformemente
contnua.
Soluc
ao:
Sabemos que dist(x0 , A) = inf {kx0 ak}.
aA

Entao, dist(x0 , A) kx0 xk kx0 y0 k + ky0 xk, isto e


dist(x0 , A) kx0 y0 k ky0 xk
assim temos que dist(x0 , A)kx0 y0 k e a menor cota inferior do conjunto {ky0 xk :
x A}. Portanto dist(x0 , A) kx0 y0 k dist(y0 , A), isto e
dist(x0 , A) dist(y0 , A) kx0 y0 k
Similarmente, dist(y0 , A) dist(x0 , A) kx0 y0 k.
Portanto, |dist(x0 , A) dist(y0 , A)| = kx0 y0 k para quaisquer x0 , y0 X.
33. Sejam K, F X, em que X e um espaco normado, K um conjunto compacto e
F um fechado. Defina dist(K, F ) = inf{kk f k : k K, f F }. Mostre que,
se K F = , entao dist(K, F ) > 0. (Note que, em particular, podemos tomar
K = {x0 }, para x0 X.)
Soluc
ao:
Seja f : K R definida por f (x) = dist(x, F ).
Como f e contnua no compacto K, ela atinge seu minimo em K, isto e existe p K
tal que min f (x) = f (p) = M .
xK

Seja A = {x X : 0 dist(x, p) M }.
Supor que M = 0. Entao A = {p} p F , mas F e fechado, p F . Assim temos
que F K 6= ()
34. Sejam K, F X, em que X e um espaco normado de dimensao finita, K um
subconjunto compacto e F um fechado. Mostre que existem k0 K e f0 F tais
que dist(K, F ) = kk0 f0 k.
Soluc
ao:
Para cada n , existe kn K e fn F tal que:
dist(K, F ) kkn fn k dist(K, F ) + 1/n
2

Ja que K e compacto, temos que kni k0 K. Para > 0 pode-se afirmar que
para i suficientemente grande
kk0 fni k kk0 kni k + kkni fni k dist(K, F ) +
dist(K,F )+ e compacto (pois dimX < ) e fn F , fn f0 F = F .
Agora F B
i
i
Entao
dist(K, F ) kk0 f0 k = lim kkni fni k lim (dist(K, F ) + 1/ni ) = dist(K, F )
i

Assim temos que dist(K, F ) = kk0 f0 k.


52. Sejam X, Y espacos normados. Mostre:
(a) Tn T em implica Tn x T x para todo x X;
(b) Se X tiver dimensao finita, entao Tn x T x para todo x X implica Tn T
em L(X, Y );
(c) De um exemplo mostrando que, em espacos de dimensao infinita, (b) pode ser
falso.
(d) Sejam X, Y, Z espacos normados. Se S L(Y, Z) e T L(X, Y ), mostre que
S T = ST L(X, Z) e kST k kSkkT k.
(e) Sejam X, Y, Z espacos normados, com Y, Z completos. Suponha que Sn S
em L(X, Y ) e Tn T em L(Y, Z). Mostre que Tn Sn T S L(X, Z).
Soluc
ao:
(a) Como Tn T , dado > 0 existe n0 N tal que se n n0 , kTn T k < . Logo
kTn x T xk = k(Tn T )xk kTn T kkxk < kxk, x X
(b) Seja = {x1 , . . . , xn } uma base para X.
Dado x X, temos que x = 1 x1 + . . . + n xn . Logo existe c > 0 tal que
kxk = k1 x1 + . . . + n xn k c(|1 | + . . . + |n |). Entao para todo x X
pode-se achar C > 0 tal que kxk C.
Como Tn x T x x X, dado > 0 existe N N tal que se n N ,
kTn x T xk < . Logo
kTn x T xk

<
<
kxk
kxk
C
Assim temos que kTn T k < isto e Tn T . So falta mostrar que T L(X, Y ).
De fato
kT xk =

kT x Tn x + Tn xk
kT x Tn xk + kTn k
+ kTn kkxk
( + kTn k)kxk, kxk = 1
3

(c) Seja X = l = {x = (i ) :
Entao T = lim Tn = 0.

|i |2 < } e Tn (x) = (0, . . . , 0, n , n+1 , . . .).

i=1

Assim para en = (0, . . . , 0, 1, 0, . . .) temos que kTn en k = 1, kTn k = 1. Logo


kTn T k = kTn k = 1.
(d) T : X Y , S : Y Z. Mostremos que S T = ST e linear e contnua.
Linearidade
Seja x1 , x2 X e K. Logo
(S T )(x1 + x2 ) =
=
=
=

S(T (x1 + x2 )), linearidade de T


S(T x1 + T x2 ), linearidade de S
S(T x1 ) + S(T x2 )
(S T )(x1 ) + (S T )(x2 )

Continuidade
k(S T )(x)k = kS(T (x))k kSkkT (x)k kSkkT kkxk
portanto kST k kSkkT k.
(e)
kTn Sn T Sk =

kTn Sn Tn S + Tn S T Sk
kTn Sn Tn Sk + kTn S T Sk
kTn (Sn S)k + k(Tn T )Sk
kTn k kSn Sk + kTn T k kSk
| {z } | {z }
0

Portanto Tn Sn T S.
53. Sejam X espaco normado e A X um conjunto arbitrario. Suponha que S A
seja denso em A e f : S Y uma aplicacao uniformemente contnua, sendo Y um
espaco de Banach. Mostre que existe uma u
nica extensao contnua F : A Y de f ,
a qual e uniformemente contnua. Se f : S Y for linear (isso implica que S e um
subespaco de A), mostre que F : A Y e linear. Esse resultado, no caso em que f
e linear, e conhecido como Teorema da Extensao Limitada.
Soluc
ao:
Unicidade: Supor que F e G sao dois extensoes para f de S em A.
Fixado a A, queremos mostrar que F (a) = G(a). Se a S e obvio pela definicao
de extensao. Suponhamos que a S = A, entao pela caracterizacao da densidade
existe uma sequencia (sn ) S com sn a. Agora
F (a) = lim F (sn ) = lim f (sn ) = lim G(sn ) = G(a)
n

assim temos que F (a) = G(a).


Existencia: Mostraremos que a A o limite de f (b) existe quando b a em S.
4

Este limite vai ser F (a).


Agora fixemos a A e seja (sn ) S que converge em A (pois S e denso em A).
Afirmacao: (f (sn )) e uma sequencia de Cauchy.
Seja lim f (sn ) = L. Para mostrar que lim
f (s) existe temos que mostrar que se (s0n )
sa
n

sS

e alguma outra sequencia em S que converge para a entao lim f (s0n ) = L.


n

De fato: Supor que lim f (s0n ) = L0 . Entao sn s0n a a = 0. Dado > 0, como
n
f e uniformemente contnua em S, escolhemos > 0 tal que
ks s0 k <

kf (s) f (s0 )k , para quasquier s, s0 S.

(1)

Do fato que sn s0n 0 escolhemos n N tal que


n N, ksn s0n k <

kf (sn ) f (s0n )k <

assim temos que f (sn ) f (s0n ) 0. Mas f (sn ) f (s0n ) L L0 , pela unicidade do
limite temos que L = L0 .
Para mostrar a afirmacao. Dado > 0 e f uniformemente contnua escolhemos > 0
tal que (1) seja satisfeito. Como sn e uma sequencia de Cauchy, pode-se achar n N
tal que:
n, m N, ksn s0n k < kf (sn ) f (s0n )k <
assim f (sn ) e uma sequencia de Cauchy e portanto convergente.
Agora seja F a extenao de f sobre A, definamos
F (a) = lim
f (s), a A
sa
sS

F assim definida e uniformemente contnua.


Dado > 0 achemos > 0 tal que
a, a0 A e ka a0 k <

kf (a) f (a0 )k <

de novo escolhemos > 0 tal que (1) seja satisfeito.


Fixemos a e a0 em A com ka a0 k < . Escolhemos (sn ) e (s0n ) de S com sn s e
s0n s0 . Entao podemos escolher N N tal que
n N, ksn s0n k <
agora da definicao de F ,
kF (a) F (a0 )k = k lim f (sn ) lim f (s0n )k
n

=
=

n
k lim (f (sn ) f (s0n )k
n
lim kf (sn ) f (s0n )k
n

Assim temos que F e uniformemente contnua em A.


Se f for linear, seja x A. Entao existe (xn ) S: xn x.
Como
kf (xn ) f (xm )k = kf (xn xm )k kf k kxn xm k 0,
5

entao (f (xn )) e de Cauchy em Y e, portanto, converge a y Y (pois Y e de Banach).


Definamos F (x) = y. Assim definido, pode-se mostrar que a definicao nao depende
la escolha da sequencia que converge para x.
De fato, se xn x e zn x, entao (x1 , z1 , x2 , z2 , . . .) tambem converge para x e,
como f e limitado, a sequencia das imagens tambem converge. Portanto, as dois
subsequencias (f (xn )) e (f (zn )) convergem para o mesmo limite.
Entao:
F e linear (e claro).
F (x) = f (x), x S.
Como kf (xn )k kf k kxn k e f (xn ) y = F (x), se xn x, entao kF (x)k
kf kkxk. Portanto, kF k kf k. Mas, o fato de F ser extensao de f , kF k kf k,
entao kf k = kF k.
55. Mostre que o espaco CL1 ([a, b], R) das funcoes contnuas f : [a, b] R com a norma
k kL1 nao e completo. Mostre tambem que CL2 ([a, b], R) com a norma k kL2 nao e
completo.
Soluc
ao:
Para mostrar que CL1 ([a, b], k kL1 ) nao e completo seja fn CL1 ([1, 1], k kL1 )
definido por

1 , x [1, n1 ]
nx , x [ n1 , n1 ]
fn (x) =

1 , x [ n1 , 1]
Seja n > m. Notar que |fn (x) fm (x)| e smetrica en torno de x = 0. Usando isto
temos que:

nx mx , x [0, n1 ]
1 mx , x [ n1 , m1 ]
|fn (x) fm (x)| =

0
, x [ m1 , 1]
Assim temos que:
Z

1
n

Z
|fn (x) fm (x)|dx =

1
m

Z
(n m)x dx +

1 mx dx

1
n

1
1
1 m 1
n 1
= (n m) 2 +

+
2
2n  m n
2m
2 n2

1 1
1
=

2 m n
Usando a smetria temos kfn fm kL1 = m1 n1 . Seja N > 2 . Entao para m, n N ,
temos que
1
1

kfn fm kL1 < +
< + =
n m
2 2
assim a sequencia e de Cauchy com a norma k kL1 . No entanto, o limite nao e

contnuo. Os calculos mostram que para a funcao definida por

1 , x [1, 0)
0 , x=0
f (x) =

1 , x (0, 1]
temos kf fn kL1 = n1 , e assim
lim kf fn kL1 = 0

Portanto a sequencia de funcoes (fn ), fn CL1 ([1, 1], k kL1 ) converge para uma
funcao descontnua sob a norma kkL1 , consequentemente o espaco CL1 ([1, 1], kkL1 )
nao e completo.
Para mostrar que CL2 ([a, b], kkL2 ) nao e completo seja fn CL2 ([0, 1], kkL2 ) definido
por

0
, x [0, 21 ]

n(x 21 ) , x [ 21 , 12 + n1 ]
fn (x) =

1
, x [ 12 + n1 , 1]
Assim temos que para n > m,

1
fm
fn

1
2

1
2

+ n1

1
2

+ m1

kfn fm k

|fn (x) fm (x)|2 dx

=
0

Z
=

:0

 Z 21 + n1


2
|fn (x)
fm (x)| dx +
1

1
2

0

|fn (x) fm (x)|2 dx

Z
+

1
1
+m
2

|fn (x) fm (x)|2 dx +

1
1
+n
2
1
1
+n
2

Z
=

:0




2


|f (x)
 fm (x)| dx

n

1 
1 
+m
2


2
Z 1+ 1 
2
m
1
m 2
(n m) x
1 mx +
dx +
dx
1
1
2
2
+
2
n

1
2

(n m)2 2n3 6mn2 + 6m2 n 2m3


+
3n3
6mn3
1
1
(n m)2
<

=
2
3mn
3m 3n
q
1
1
Logo kfn fm kL2 < 3m
3n
. Seja N > 322 . Entao para m, n N , temos que
=

r
kfn fm kL2 <

1
1

<
3m 3n

1
1
+
<
3m 3n

2 2
+
=
2
2

assim a sequencia e de Cauchy com a norma k kL2 . No entanto, o limite nao e


contnuo. Os calculos mostram que para a funcao definida por

0 , x [0, 21 ]
f (x) =
1 , x ( 21 , 1]
temos kf fn k2L2 =

1
,
3n

e assim
lim kf fn kL2 = 0

Portanto a sequencia de funcoes (fn ), fn CL2 ([0, 1], k kL2 ) converge para uma
funcao descontnua sob a norma k kL2 , consequentemente o espaco CL2 ([0, 1], k kL2 )
nao e completo.
Exerccios optativos
36. De um exemplo de um espaco de Banach X que possui um subespaco fechado Y
1 (0) X tal que dist(x, Y ) = 1. (Compare com o
de modo que nao exista x B
Exerccio 35.)
Soluc
ao:
Seja E o subespaco deR(C[0, 1], R) com a norma k k das funcoes tais que f (0) = 0.
1
Seja S = {f E : 0 f (t)dt = 0}. S assim definido e um subespaco de E. S
e fechado j
Ra1 que se (fn ) S e uma sequencia convergindo uniformemente para f ,
segue que 0 f (t)dt = 0, isto e f S.
8

1 (0) tal que dist(f, S) = 1. Entao teriamos que kf gk 1


Supor que temos f B
g S.
Tomemos em particular a fun
por gn (t) = t1/n , 0 t 1, n Z+ .
Rc1ao gn E dada
n
.
Entao temos que kgn k = 1 e 0 gn (t)dt = n+1
O vetor


Z
n+1 1
h=f
f (t)dt gn S.
n
0
Logo
n+1
1 kf hk =
n

f (t)dt.
0

Como isto vale para todo n inteiro positivo segue dai que
Z 1
f (t)dt 1
0

o que e um absurdo pois como f (t) 1 (0 t 1) e f (0) = 0 a continuidade de f


obriga que esta integral seja menor do que 1.
40. Considere o espaco de Banach X = C([0, 1], R) com a norma k k . Seja A X um
conjunto convexo completo e = inf kak. De exemplos de subconjuntos A, tais que
aA

(a) existem infinitos pontos a A tais que kak = ;


(b) existe uma sequencia (an ) em A tal que kan k , mas nenhuma de sus subsequencias e de Cauchy;
(c) nao existe a A tal que kak = .
Soluc
ao:
64. Mostre que um espaco normado X e um espaco de Banach se, e somente se, toda
serie absolutamente convergente for convergente.
Soluc
ao:
] Supor que X e completo,

X
i=1

trar a convergencia de sn =

xi e absolutamente convergente. Precisamos mosn


X

xi . Dado > 0, escolhemos N N tal que

i=1

kxi k < , entao ksn sm k < sempre que n, m N . Assim (sn ) e uma

i=N

sequencia de Cauchy, entao e convergente (pois X e completo).

[ Supor que toda sequencia absolutamente convergente converge em X.


Seja (xn ) uma sequencia de Cauchy. Para mostrar que X e completo, devemos
mostrar que a sequencia (xn ) converge em X.
Para cada j 1 pode-se achar nj tal que
kxn xm k 2j
para todo n, m nj . Defina v1 = xn1 e vj = xnj xnj1 para j 2. Assim
temos que
kvj k 2j .

X
Entao a serie
vj e absolutamente convergente e pela hipotese e convergente.
j=1

Logo a suma parcial,


sj =

vi

ij

e igual a xnj . Ja que

vj e convergente, temos que sj s. Em outras

j=1

palabras, xnj s. Entao a subsequencia da sequencia de Cauchy (xn ) converge.


Usando o item (c) do exercicio 20, temos que xn s.
65. Sejam : N N uma bijecao e
yi = x(i) . Mostre que
serie

xi uma serie absolutamente convergente. Defina

i=1

yn e absolutamente convergente e

i=1

X
i=1

yi =

xi , isto e, a

i=1

yi e comutativamente convergente.

i=1

Soluc
ao:
Ja que e bijetiva, temos que:
{y1 , y2 , . . . , yn , . . .} = {x1 , x2 , . . . , xm , . . .}
entao

kyi k =

i=1

kxi k <

i=1

n

X



Seja s =
xi . Dado > 0 existe n N tal que
xi s < .


i=1
i=1
Agora {1, 2, . . . , n} {(1), (2), . . . , (n())} para algum n() N. Logo para
m n() temos que:


m
X



x(i) s <


i=1

Assim

X
i=1

x(i) converge para s, isto e

X
i=1

10

yi =

X
i=1

xi

Вам также может понравиться